XAT Trignometry Questions PDF [Most Important]

0
202
Trignometry Questions PDF
Trignometry Questions PDF

XAT Trignometry Questions PDF [Most Important]

Download Trignometry Questions for XAT PDF – XAT Trignometry questions pdf by Cracku. Top 12 very Important Trignometry Questions for XAT based on asked questions in previous exam papers.

Download Trignometry Questions for XAT

Enroll to XAT 2023 Crash Course

Question 1: If $\frac{\sin\theta+\cos\theta}{\sin\theta-\cos\theta}=5$, then the value of $\frac{4\sin^2\theta+3}{2\cos^2\theta+2}$ is:

a) $\frac{75}{17}$

b) $\frac{75}{34}$

c) $\frac{1}{2}$

d) $\frac{3}{2}$

1) Answer (B)

Solution:

$\frac{\sin\theta+\cos\theta}{\sin\theta-\cos\theta}=5$

$\sin\theta+\cos\theta=5\sin\theta\ -5\cos\theta\ $

$4\sin\theta=6\cos\theta\ $

$\tan\theta\ =\frac{3}{2}$

$\sec\theta\ =\sqrt{\left(\frac{3}{2}\right)^2+1}=\frac{\sqrt{13}}{2}$

$\cos\theta\ =\frac{2}{\sqrt{13}}$

$\sin\theta\ =\sqrt{1-\left(\frac{2}{\sqrt{13}}\right)^2}=\frac{3}{\sqrt{13}}$

$\frac{4\sin^2\theta+3}{2\cos^2\theta+2}=\frac{4\left(\frac{3}{\sqrt{13}}\right)^2+3}{2\left(\frac{2}{\sqrt{13}}\right)^2+2}$

$=\frac{\frac{36}{13}+3}{\frac{8}{13}+2}$

$=\frac{\frac{36+39}{13}}{\frac{8+26}{13}}$

$=\frac{75}{34}$

Hence, the correct answer is Option B

Question 2: Find the value of $\frac{\tan^2 30^\circ}{\sec^2 30^\circ} + \frac{\cosec^2 45^\circ}{\cot^2 45^\circ} – \frac{\sec^2 60^\circ}{\cosec^2 60^\circ}$

a) $-\frac{3}{4}$

b) $\frac{5}{4}$

c) $\frac{13}{4}$

d) $\frac{23}{12}$

2) Answer (A)

Solution:

$\frac{\tan^230^{\circ}}{\sec^230^{\circ}}+\frac{\operatorname{cosec}^245^{\circ}}{\cot^245^{\circ}}-\frac{\sec^260^{\circ}}{\operatorname{cosec}^260^{\circ}}=\frac{\left(\frac{1}{\sqrt{3}}\right)^2}{\left(\frac{2}{\sqrt{3}}\right)^2}+\frac{\left(\sqrt{2}\right)^2}{\left(1\right)^2}-\frac{\left(2\right)^2}{\left(\frac{2}{\sqrt{3}}\right)^2}$

$=\frac{\frac{1}{3}}{\frac{4}{3}}+\frac{2}{1}-\frac{4}{\frac{4}{3}}$

$=\frac{1}{4}+2-\frac{4\times3}{4}$

$=\frac{-3}{4}$

Hence, the correct answer is Option A

Question 3: If $3 \sec \theta + 4 \cos \theta – 4\sqrt{3} = 0$ where $\theta$ is an acute angle then the value of $\theta$ is:

a) $20^\circ$

b) $30^\circ$

c) $60^\circ$

d) $45^\circ$

3) Answer (B)

Solution:

$3\sec\theta+4\cos\theta-4\sqrt{3}=0$

$\frac{3}{\cos\theta\ }+4\cos\theta-4\sqrt{3}=0$

$4\cos^2\theta-4\sqrt{3}\cos\theta\ +3=0$

$4\cos^2\theta-2\sqrt{3}\cos\theta\ -2\sqrt{3}\cos\theta+3=0$

$2\cos\theta\ \left(2\cos\theta-\sqrt{3}\right)\ -\sqrt{3}\left(2\cos\theta-\sqrt{3}\right)=0$

$\ \left(2\cos\theta-\sqrt{3}\right)\left(2\cos\theta-\sqrt{3}\right)=0$

$\ \left(2\cos\theta-\sqrt{3}\right)^2=0$

$\ 2\cos\theta-\sqrt{3}=0$

$\cos\theta=\frac{\sqrt{3}}{2}$

$\theta=30^{\circ\ }$

Hence, the correct answer is Option B

Question 4: If $3 \tan \theta = 2\sqrt{3} \sin \theta, 0^\circ < \theta < 90^\circ$, then find the value of $2 \sin^2 2\theta – 3 \cos^2 3\theta$.

a) 1

b) $\frac{3}{2}$

c) $\frac{1}{2}$

d) $-\frac{3}{2}$

4) Answer (B)

Solution:

$3\tan\theta=2\sqrt{3}\sin\theta$

$3\frac{\sin\theta\ }{\cos\theta\ }=2\sqrt{3}\sin\theta$

$\cos\theta\ =\frac{3}{2\sqrt{3}}$

$\cos\theta\ =\frac{\sqrt{3}}{2}$

$\theta\ =30^{\circ\ }$ [$0^\circ < \theta < 90^\circ$]

$2\sin^22\theta-3\cos^23\theta=2\sin^260^{\circ\ }-3\cos^290^{\circ\ }$

= $2\left(\frac{\sqrt{3}}{2}\right)^2-3\left(0\right)^2$

= $\frac{3}{2}$

Hence, the correct answer is Option B

Question 5: If $\sin^2 \theta – \cos^2 \theta – 3 \sin \theta + 2 = 0, 0^\circ < \theta < 90^\circ$, then what is the value of $\frac{1}{\sqrt{\sec \theta – \tan \theta}}$ is:

a) $\sqrt[4]{3}$

b) $\sqrt[2]{2}$

c) $\sqrt[2]{3}$

d) $\sqrt[4]{2}$

5) Answer (A)

Solution:

$\sin^2\theta-\cos^2\theta-3\sin\theta+2=0$

$\sin^2\theta-\left(1-\sin^2\theta\ \right)-3\sin\theta+2=0$

$2\sin^2\theta-3\sin\theta+1=0$

$2\sin^2\theta-2\sin\theta-\sin\theta\ +1=0$

$2\sin\theta\ \left(\sin\theta\ -1\right)-1\left(\sin\theta\ -1\right)=0$

$\left(\sin\theta\ -1\right)\left(2\sin\theta\ -1\right)=0$

$\sin\theta\ -1=0$  or  $2\sin\theta\ -1=0$

$\sin\theta\ =1$  or  $\sin\theta\ =\frac{1}{2}$

$\theta\ =90^{\circ\ }$  or  $\theta\ =30^{\circ\ }$

Given, $0^\circ < \theta < 90^\circ$

$\Rightarrow$  $\theta\ =30^{\circ\ }$

$\frac{1}{\sqrt{\sec\theta-\tan\theta}}=\frac{1}{\sqrt{\sec30^{\circ\ }-\tan30^{\circ\ }}}$

$=\frac{1}{\sqrt{\frac{2}{\sqrt{3}}\ -\frac{1}{\sqrt{3}}}}$

$=\frac{1}{\sqrt{\frac{1}{\sqrt{3}}}}$

$=\sqrt[\ 4]{3}$

Hence, the correct answer is Option A

Take XAT 2023 Mock Tests

Question 6: Find the value of $\sin^2 60^\circ + \cos^2 30^\circ – \sin^2 45^\circ – 3 \sin^2 90^\circ$.

a) $\frac{1}{3}$

b) $-1\frac{3}{4}$

c) $-2\frac{1}{2}$

d) $-2$

6) Answer (D)

Solution:

$\sin^260^{\circ}+\cos^230^{\circ}-\sin^245^{\circ}-3\sin^290^{\circ}=\left(\frac{\sqrt{3}}{2}\right)^2+\left(\frac{\sqrt{3}}{2}\right)^2-\left(\frac{1}{\sqrt{2}}\right)^2-3\left(1\right)^2$

$=\frac{3}{4}+\frac{3}{4}-\frac{1}{2}-3$

$=\frac{3+3-2-12}{4}$

$=\frac{-8}{4}$

$=-2$

Hence, the correct answer is Option D

Question 7: The value of $\frac{\sec^2 60^\circ \cos^2 45^\circ + \cosec^2 30^\circ}{\cot 30^\circ \sec^2 45^\circ – \cosec^2 30^\circ \tan 45^\circ}$ is:

a) $-3(2 + \sqrt{3})$

b) $3(2 – \sqrt{3})$

c) $-3(2 – \sqrt{3})$

d) $3(2 + \sqrt{3})$

7) Answer (A)

Solution:

$\frac{\sec^260^{\circ}\cos^245^{\circ}+\operatorname{cosec}^230^{\circ}}{\cot30^{\circ}\sec^245^{\circ}-\operatorname{cosec}^230^{\circ}\tan45^{\circ}}=\frac{\left(2\right)^2.\left(\frac{1}{\sqrt{2}}\right)^2+\left(2\right)^2}{\left(\sqrt{3}\right)\left(\sqrt{2}\right)^2-\left(2\right)^2.\left(1\right)}$

$=\frac{4\times\frac{1}{2}+4}{2\sqrt{3}-4}$

$=\frac{6}{2\sqrt{3}-4}$

$=\frac{3}{\sqrt{3}-2}$

$=\frac{3}{\sqrt{3}-2}\times\frac{\sqrt{3}+2}{\sqrt{3}+2}$

$=\frac{3\left(\sqrt{3}+2\right)}{3-4}$

$=-3\left(2+\sqrt{3}\right)$

Hence, the correct answer is Option A

Question 8: If $\sin^2 \theta = 2 \sin \theta – 1, 0^\circ \leq \theta \leq 90^\circ $, then find the value of: $\frac{1 + \cosec \theta}{1 – \cos \theta}$.

a) -2

b) 1

c) 2

d) -1

8) Answer (C)

Solution:

$\sin^2\theta=2\sin\theta-1$

$\sin^2\theta-2\sin\theta+1=0$

$\left(\sin\theta-1\right)^2=0$

$\sin\theta-1=0$

$\sin\theta=1$

$0^{\circ}\le\theta\le90^{\circ}$

$\Rightarrow$  $\theta=90^{\circ}$

$\frac{1+\operatorname{cosec}\theta}{1-\cos\theta}=\frac{1+\operatorname{cosec}90^{\circ\ }}{1-\cos90^{\circ\ }}$

$=\frac{1+1\ }{1-0\ }$

$=2$

Hence, the correct answer is Option C

Question 9: Simplify $\sec^2 \alpha \left(1 + \frac{1}{\cosec \alpha}\right)\left(1 – \frac{1}{\cosec \alpha}\right)$.

a) $\tan^4 \alpha$

b) $\sin^2 \alpha$

c) 1

d) -1

9) Answer (C)

Solution:

$\sec^2\alpha\left(1+\frac{1}{\operatorname{cosec}\alpha}\right)\left(1-\frac{1}{\operatorname{cosec}\alpha}\right)=\frac{1}{\cos^2\alpha}\left(1+\sin\alpha\right)\left(1-\sin\alpha\right)$

$=\frac{1}{\cos^2\alpha}\left(1-\sin^2\alpha\right)$

$=\frac{1}{\cos^2\alpha}\left(\cos^2\alpha\right)$

$=1$

Hence, the correct answer is Option C

Question 10: In $\triangle$ABC, right angled at B, if cot A = $\frac{1}{2}$, then the value of $\frac{\sin A(\cos C + \cos A)}{\cos C(\sin C – \sin A)}$ is

a) 3

b) -3

c) -2

d) 2

10) Answer (B)

Solution:

cot A = $\frac{\text{Adjacent side}}{\text{Opposite side}}$ $\frac{1}{2}$

$\frac{\sin A(\cos C + \cos A)}{\cos C(\sin C – \sin A)}$ = $\frac{\frac{2}{\sqrt{5}}\left(\frac{2}{\sqrt{5}}+\frac{1}{\sqrt{5}}\right)}{\frac{2}{\sqrt{5}}\left(\frac{1}{\sqrt{5}}-\frac{2}{\sqrt{5}}\right)}$

= $\frac{\left(\frac{3}{\sqrt{5}}\right)}{\left(-\frac{1}{\sqrt{5}}\right)}$

= -3

Hence, the correct answer is Option B

Question 11: If $\tan \theta + 3 \cot \theta – 2\sqrt{3} = 0, 0^\circ < \theta < 90^\circ$, then what is the value of $(\cosec^2 \theta + \cos^2 \theta)?$

a) $\frac{2}{3}$

b) $\frac{19}{12}$

c) $\frac{14}{3}$

d) $\frac{11}{12}$

11) Answer (B)

Solution:

$\tan\theta+3\cot\theta-2\sqrt{3}=0$

$\tan\theta+\frac{3}{\tan\theta\ }-2\sqrt{3}=0$

$tan^2\theta-2\sqrt{3}\tan\theta\ +3=0$

$\left(\tan\theta\ -\sqrt{3}\right)^2=0$

$\tan\theta\ -\sqrt{3}=0$

$\tan\theta\ =\sqrt{3}$

$0^\circ < \theta < 90^\circ$

$\Rightarrow$  $\theta\ =60^{\circ\ }$

$\operatorname{cosec}^2\theta+\cos^2\theta=\operatorname{cosec}^260^{\circ\ }+\cos^260^{\circ\ }$

= $\left(\frac{2}{\sqrt{3}}\right)^2\ +\left(\frac{1}{2}\right)^2$

= $\frac{4}{3}\ +\frac{1}{4}$

= $\frac{16+3}{12}$

= $\frac{19}{12}$

Hence, the correct answer is Option B

Question 12: If $\sin \alpha + \sin \beta = \cos \alpha + \cos \beta = 1$, then $\sin \alpha + \cos \alpha =$?

a) -1

b) 0

c) 1

d) 2

12) Answer (C)

Solution:

$\sin\alpha+\sin\beta=1$

$\sin^2\alpha+\sin^2\beta+2\sin\alpha\ \sin\beta\ =1$……(1)

$\cos\alpha+\cos\beta=1$

$\cos^2\alpha+\cos^2\beta+2\cos\alpha\ \cos\beta\ =1$……(2)

Adding (1) and (2),

$\left(\sin^2\alpha\ +\cos^2\alpha\right)+\left(\sin^2\beta\ +\cos^2\beta\right)+2\sin\alpha\ \sin\beta\ +2\cos\alpha\ \cos\beta\ =1+1$

$1+1+2\sin\alpha\ \sin\beta\ +2\cos\alpha\ \cos\beta\ =2$

$2\left[\cos\alpha\ \cos\beta+\sin\alpha\ \sin\beta\right]=0$

$\cos\left(\beta-\alpha\right)=0$

$\beta-\alpha=90^{\circ\ }$

$\beta\ =90^{\circ\ }+\alpha\ $

$\sin\alpha+\sin\beta=1$

$\sin\alpha+\sin\left(90^{\circ}-\alpha\ \right)=1$

$\sin\alpha+\cos\alpha=1$

Hence, the correct answer is Option C

Question 13: Find the value of $\operatorname{cosec}(60^{\circ}+A)-\sec(30^{\circ}-A)+\frac{\operatorname{cosec}49^{\circ}}{\sec41^{\circ}}$.

a) 1

b) 0

c) -1

d) 2

13) Answer (A)

Solution:

$\operatorname{cosec}(60^{\circ}+A)-\sec(30^{\circ}-A)+\frac{\operatorname{cosec}49^{\circ}}{\sec41^{\circ}}$

= $\operatorname{cosec}(60^{\circ}+A)-\sec(90^{\circ}-60^{\circ}-A)+\frac{\operatorname{cosec}49^{\circ}}{\sec\left(90-49\right)^{\circ}}$

$\left[\sec\left(90\ -\theta\right)=\operatorname{cosec}\theta\right]$

= $\operatorname{cosec}\left(60^{\circ}+A\right)-\sec\left(90^{\circ}-\left(60^{\circ}+A\right)\right)+\frac{\operatorname{cosec}49^{\circ}}{\operatorname{cosec}49^{\circ}}$

= $\operatorname{cosec}\left(60^{\circ}+A\right)-\operatorname{cosec}\left(60^{\circ}+A\right)+1$

= 1

Hence, the correct answer is Option A

Question 14: If $\frac{1}{1 – \sin \theta} + \frac{1}{1 + \sin \theta} = 4 \sec \theta, 0^\circ < \theta < 90^\circ$, then the value of $\cot\theta+\operatorname{cosec}\theta$ is:

a) $\frac{4\sqrt{3}}{3}$

b) $\sqrt{3}$

c) $\frac{5\sqrt{3}}{3}$

d) $3\sqrt{3}$

14) Answer (B)

Solution:

$\frac{1}{1-\sin\theta}+\frac{1}{1+\sin\theta}=4\sec\theta$

$\frac{1+\sin\theta\ +1-\sin\theta\ }{1-\sin^2\theta}=\frac{4}{\cos\theta}$

$\frac{2}{\cos^2\theta}=\frac{4}{\cos\theta}$

$\cos\theta=\frac{1}{2}$

$0^\circ < \theta < 90^\circ$

$\Rightarrow$  $\theta=60^{\circ}$

$\cot\theta+\operatorname{cosec}\theta=\cot60^{\circ}+\operatorname{cosec}60^{\circ}$

= $\frac{1}{\sqrt{3}}+\frac{2}{\sqrt{3}}$

= $\frac{3}{\sqrt{3}}$

= $\sqrt{3}$

Hence, the correct answer is Option B

Question 15: $(\operatorname{cosec}A-\cot A)(1+\cos A)=?$

a) $\cos A$

b) $\sin A$

c) $\cot A$

d) $\cosec A$

15) Answer (B)

Solution:

$(\operatorname{cosec}A-\cot A)(1+\cos A)=\left(\frac{1}{\sin A}-\frac{\cos A}{\sin A}\right)\left(1+\cos A\right)$

= $\left(\frac{1-\cos A}{\sin A}\right)\left(1+\cos A\right)$

= $\left(\frac{1-\cos^2A}{\sin A}\right)$

= $\frac{\sin^2A}{\sin A}$

= $\sin A$

Hence, the correct answer is Option B

Enroll to XAT 2023 Crash Course

LEAVE A REPLY

Please enter your comment!
Please enter your name here